组合数学 (Fall 2023)/Problem Set 2: Difference between revisions

From TCS Wiki
Jump to navigation Jump to search
Gispzjz (talk | contribs)
Roundgod (talk | contribs)
Line 9: Line 9:


Let <math>S=\{P_1,...,P_n\}</math> be a set of properties, and let <math>f_k</math> (respectively, <math>f_{\geq k}</math>) denote the number of objects in a finite set <math>U</math> that have '''exactly''' <math>k</math> (respectively, '''at least''' <math>k</math>) of the properties.  
Let <math>S=\{P_1,...,P_n\}</math> be a set of properties, and let <math>f_k</math> (respectively, <math>f_{\geq k}</math>) denote the number of objects in a finite set <math>U</math> that have '''exactly''' <math>k</math> (respectively, '''at least''' <math>k</math>) of the properties.  
Let <math>A_i</math> denote the set of objects satisfies <math>P_i</math> in <math>U</math>, for any <math>I\subseteq\{1,...,n\}</math>, we denote <math>A_I=\bigcap_{i\in I}A_i</math> with the convention that <math>A_\emptyset=U</math>.
Let <math>A_i</math> denote the set of objects that satisfy <math>P_i</math> in <math>U</math>, for any <math>I\subseteq\{1,...,n\}</math>, we denote <math>A_I=\bigcap_{i\in I}A_i</math> with the convention that <math>A_\emptyset=U</math>.
Show that
Show that



Revision as of 12:52, 13 April 2023

Problem 1

Problem 2

Give [math]\displaystyle{ n,m,k }[/math], find the number of integer solutions to the following equation: [math]\displaystyle{ a_1+a_2+...+a_n=m, \forall 1\leq i\leq n, 0\leq a_i\lt k }[/math]. You should give a formula and explain your answer.

Problem 3

Let [math]\displaystyle{ S=\{P_1,...,P_n\} }[/math] be a set of properties, and let [math]\displaystyle{ f_k }[/math] (respectively, [math]\displaystyle{ f_{\geq k} }[/math]) denote the number of objects in a finite set [math]\displaystyle{ U }[/math] that have exactly [math]\displaystyle{ k }[/math] (respectively, at least [math]\displaystyle{ k }[/math]) of the properties. Let [math]\displaystyle{ A_i }[/math] denote the set of objects that satisfy [math]\displaystyle{ P_i }[/math] in [math]\displaystyle{ U }[/math], for any [math]\displaystyle{ I\subseteq\{1,...,n\} }[/math], we denote [math]\displaystyle{ A_I=\bigcap_{i\in I}A_i }[/math] with the convention that [math]\displaystyle{ A_\emptyset=U }[/math]. Show that

[math]\displaystyle{ f_k=\sum_{i=k}^n(-1)^{i-k}\binom{i}{k}g_i }[/math] and [math]\displaystyle{ f_{\geq k}=\sum_{i=k}^n(-1)^{i-k}\binom{i-1}{k-1}g_i. }[/math]

where [math]\displaystyle{ g_i=\sum_{I\subseteq\{1,...,n\},|I|=i }|A_I|. }[/math]